Exam 3 DA & Book Practice Questions

Pataasin ang iyong marka sa homework at exams ngayon gamit ang Quizwiz!

A PT w/ chronic renal failure is prescribed a protein restricted diet. Which is an incomplete protein they should avoid? nuts eggs milk fish

nuts

The nurse is caring for a patient who is newly diagnosed with advanced lung cancer. The patient says, "I don't want to die." How should the nurse respond? "I am sure you are very afraid. Tell me what you are most scared of." "Everyone is going to die sometime." "Cancer is very treatable. I am sure you'll be fine." "If you follow the treatment plan, you'll have the best chance of living."

"I am sure you are very afraid. Tell me what you are most scared of."

The nurse is reinforcing the teaching of the primary health-care provider to the patient who had a scrotal injury. Which response given by the patient indicates the need for further teaching? "I must keep the area clean and dry." "I must gently apply an ice pack on the scrotal skin." "I must report if nausea and vomiting are present." "I must use folded towels to elevate the scrotum."

"I must gently apply an ice pack on the scrotal skin."

A nurse is providing instructions to a patient who is prescribed antibiotics and phenazopyridine for a urinary tract infection. Which statement made by the patient indicates effective learning? "I should reduce my fluid intake." "I should report if my urine turns orange." "I should administer phenazopyridine for only 3 days." "I should stop using antibiotics if my symptoms disappear."

"I should administer phenazopyridine for only 3 days." (Rationale: The adminstration of phenazopyridine beyond 3 days can mask symptoms of dysuria and frequency and urgency. Phenazopyridine may turn urine orange or red which is normal)

A patient is diagnosed with benign prostatic hyperplasia (BPH) and has minimal symptoms. The primary health-care provider recommends active surveillance of the patient. What should the nurse include with discharge teaching? Select all that apply. "I should advise the patient to avoid sedatives." "I should advise the patient to avoid all over-the-counter (OTC) medications." "I should advise the patient to visit the provider yearly." "I should advise the patient to increase fluid intake until bed time." "I should advise the patient to avoid consumption of selenium-rich foods."

"I should advise the patient to avoid sedatives." "I should advise the patient to visit the provider yearly." Rationale: Watchful waiting or active surveillance is the treatment option used in patients with minimal symptoms. The patient should avoid sedatives as it may worsen symptoms. Rationale: Patients should visit the provider yearly for evaluation and to undergo a digital rectal examination.

The nurse is providing instructions to a patient with blunt testicular trauma while playing baseball. Which statement by the patient indicates the need for further teaching? "I should take my pain medications as ordered." "I should use folded towels to elevate my scrotum." "I should get adequate rest for early recovery." "I should apply ice directly to the skin of the scrotum."

"I should apply ice directly to the skin of the scrotum."

The nurse is providing dietary instructions to a patient treated for kidney stones to reduce the risk of stone formation. Which statement made by the patient indicates effective learning? "I should avoid citrus fruits." "I should reduce water intake." "I should increase salt intake in my food." "I should avoid tea, coffee, and chocolates."

"I should avoid tea, coffee, and chocolates."

A patient was diagnosed with benign prostatic hyperplasia (BPH), and the primary health-care provider decides that watchful waiting is appropriate. Which statement made by the patient requires further teaching? "I should receive yearly examinations." "I should avoid over-the-counter drugs." "I should drink lots of fluids in the evening." "I should return to the clinic if I have symptoms."

"I should drink lots of fluids in the evening." Rationale: Consuming excess fluids in the evening may worsen the symptom of nocturia. This statement requires further patient education.

A nurse is educating patients about cancer prevention. Which cancer prevention measure stated by one of the patients indicates a need for further teaching? Select all that apply. "I should include 150 minutes of vigorous activity per week." "I should ensure that as a woman I do not consume more than two drinks per day." "I should avoid exposure to known carcinogens such as tobacco." "I should try to maintain a healthy weight." "I should eat a starch-based diet to stay healthy."

"I should include 150 minutes of vigorous activity per week." "I should ensure that as a woman I do not consume more than two drinks per day." "I should eat a starch-based diet to stay healthy."

A RN has provided D/C instructions to a PT post prostatectomy for CA. Which PT statement indicates PT understanding? "I should not lift anything over 20 lbs" "To prevent dribbling, I need to limit my fluid intake to 4 cups QD" "If I see any clots in my urine, I need to call the LIP immediately" "I can begin to drive my car in 1 week"

"I should not lift anything over 20 lbs"

After undergoing an unsuccessful open-heart surgery, a patient is admitted into the hospice care unit. The nurse notices that the patient's partner appears anxious and nervous. Which statement is most appropriate for the nurse to use to provide comfort to the partner? "It is good that your partner is not on the life support system. Otherwise it could have been really bad." "Do not worry; your partner will be fine." "I understand that you are worried about your partner, do you have any questions that I can help answer?" "Pray to God so that your partner can get well soon."

"I understand that you are worried about your partner, do you have any questions that I can help answer?"

The nurse is teaching about a patient who has undergone breast surgery. Which statements indicate effective understanding? Select all that apply. "I will observe for redness, bleeding, and odor." "I will teach the patient about breast self-examination." "I will be judgmental to help create a relaxed environment." "I will avoid administering pain medications because they may cause ulcers." "I will discuss dressing changes and antiseptic cleaning techniques with the patient."

"I will observe for redness, bleeding, and odor." "I will teach the patient about breast self-examination." "I will discuss dressing changes and antiseptic cleaning techniques with the patient."

A registered nurse is teaching a patient about palliative care (PC). Which information about PC stated by the patient indicates a need for further teaching? "It focuses on relief of pain and stress associated with a severe illness." "It is provided in hospital, outpatient, and community settings." "It focuses on the care of terminally ill patients with less than 6 months to live." "It is given simultaneously with curative treatment that meets the patient's goals."

"It focuses on the care of terminally ill patients with less than 6 months to live."

While preparing a patient suspected of having esophageal cancer for a biopsy, the patient asks the nurse, "What is the difference between a benign tumor and a malignant tumor?" Which statement made by the nurse is correct? "Benign tumors undergo metastasis." "Malignant tumors are regulated by contact inhibition." "Benign tumors do not have anchorage dependence." "Malignant tumors directly invade adjacent tissues and spread to distant tissues."

"Malignant tumors directly invade adjacent tissues and spread to distant tissues."

Which statement made by a patient is an example of "Cultural Aspects of Care" domain of palliative care? Select all that apply. "I am anxious because I don't know what will happen to my family after my death." "Our community does not believe in blood transfusion." "I am suffering because God is angry with me." "Please do not include nonvegetarian items in my diet as it is against my tradition." "In our community, we do not burden children with medical information."

"Our community does not believe in blood transfusion." "Please do not include nonvegetarian items in my diet as it is against my tradition." "In our community, we do not burden children with medical information."

An older female client tells the nurse she has been experiencing new onset urinary incontinence. What is the nurse's best response? "That is not a part of the aging process." "Underpads are available to protect your clothing." "Please tell me about your symptoms." "There are surgical repair options available to help."

"Please tell me about your symptoms."

A patient is undergoing treatment for stress incontinence. Which instruction provided by the nurse is most appropriate to manage this condition? "Avoid coffee and cigarettes." "Use diapers while traveling." "Practice pelvic floor exercises regularly." "Avoid taking large amount of fluids at once."

"Practice pelvic floor exercises regularly."

A patient is undergoing treatment for stress incontinence. Which instruction provided by the nurse is most appropriate to manage this condition? "Avoid coffee and cigarettes." "Use diapers while traveling." "Practice pelvic floor exercises regularly." "Avoid taking large amount of fluids at once."

"Practice pelvic floor exercises regularly." (Rationale: Pelvic floor exercises such as Kegel exercises should be practiced to manage stress incontinence because they help to improve the strength of the pelvic floor muscles.)

A patient diagnosed with urolithiasis is prescribed tamsulosin. Which instruction is most important for the patient in this condition? "Follow a low-sodium diet." "Avoid chocolate and nuts." "Rise slowly from a seated position." "Do not worry if your urine is orange- or red-colored."

"Rise slowly from a seated position."

The nurse is teaching a nursing student the medication therapy of menopausal symptoms. Which statement made by the nursing student indicates a need for further learning? "Estrogen therapy is most effective for treating the symptoms." "Black cohosh is the most commonly used herbal treatment for menopausal symptoms." "Selective norepinephrine reuptake inhibitors are prescribed for patients who choose not to take estrogen therapy." "Selective serotonin reuptake inhibitors (SSRIs) are prescribed for patients on tamoxifen therapy for breast cancer."

"Selective serotonin reuptake inhibitors (SSRIs) are prescribed for patients on tamoxifen therapy for breast cancer."

The nurse is caring for a patient with stage II cancer who asks, "What does stage II cancer mean?" How should the nurse reply? "Stage II cancer includes large or locally invasive tumors." "Stage II cancer includes cancers that have metastasized." "Stage II cancer is more localized and contained." "Stage II cancer involves invasion of deeper tissues or local lymph nodes."

"Stage II cancer involves invasion of deeper tissues or local lymph nodes."

The nurse is talking with a patient who says his friend has a procedure where hot water is used for causing coagulation necrosis of the prostate tissue. The patient says, "That's just crazy, I don't believe him." How should the nurse respond? "It sounds like your friend is talking about an open prostatectomy." "A transurethral needle ablation includes the use of hot water." "That would be water-induced thermotherapy (WIT)." "Yes, that's a transurethral ethanol ablation of the prostate (TEAP)."

"That would be water-induced thermotherapy (WIT)." Rationale: Water-induced thermotherapy (WIT) is a 45-minute procedure performed using topical anesthesia. Hot water is used to cause coagulation necrosis of the prostatic tissue that will compress the urethra.

The nurse is caring for a patient with severe menopausal symptoms. Which reports are documented by the nurse if the condition is treated effectively? Select all that apply. "The patient has relief from hot flashes." "The patient has a normal blood pressure." "The patient has stable weight maintenance." "The patient has a normal urinating schedule." "The patient has normal pelvic pressure."

"The patient has relief from hot flashes." "The patient has a normal blood pressure." "The patient has stable weight maintenance."

The nurse is teaching a group of men with benign prostatic hyperplasia about different surgical management procedures. Which statement by the nurse indicates effective teaching? "Open prostatectomies may be used for men taking anticoagulants." "Transurethral microwave therapy causes shrinkage of the prostate gland." "Prostatic stents can be used for the long-term treatment." "A transurethral incision of the prostate involves the removal of the entire outer layer of the prostate."

"Transurethral microwave therapy causes shrinkage of the prostate gland."

Which patient is at the highest risk of developing a urinary tract infection (UTI)? 65-year-old male patient 25-year-old pregnant patient 20-year-old sexually inactive patient 15-year-old male patient with poor hygiene

25-year-old pregnant patient

A client with benign prostatic hyperplasia asks about medications that can help shrink the prostate tissue. Which type of medication should the nurse mention? 5-alpha reductase inhibitors Diuretic Chemotherapy agents Beta blockers

5-alpha reductase inhibitors

A 28-year-old male patient with a history of diabetes is diagnosed with a urinary tract infection (UTI)? Which treatment option is most suitable for the patient? Colporrhaphy 3-day course of ciprofloxacin Transurethral resection of the prostate 7-day course of trimethoprim/sulfamethoxazole

7-day course of trimethoprim/sulfamethoxazole

The nurse is comparing patients who have a normal prostate cancer risk based on prostate-specific antigen (PSA) levels. Which patient would the nurse expect to be normal? A 48-year-old man with a PSA level of 4 to 10 mg/mL A 45-year-old man with a PSA level > 10 ng/mL A 73-year-old man with a PSA level of 5 mg/mL A 50-year-old man with a PSA level of 3.9 mg/mL

A 73-year-old man with a PSA level of 5 mg/mL

Which client should the nurse identify as being at the greatest risk for developing cancer? A client diagnosed with influenza A client pregnant with her first child A client who tests positive for sickle cell anemia A client who tests positive for human papillomavirus virus

A client who tests positive for human papillomavirus virus

A PT asks the RN about the risk factors of prostate CA. Which statement indicates a need for further education? Men aged 50 and older should be monitored w/ a yearly DRE exposure to and handling cadmium batteries A high fat diet will assist in preventing this type of CA African-American men have a high incidence of this type of cancer

A high fat diet will assist in preventing this type of CA

Which patient is at risk of renal trauma? A patient with hypertension A patient who smokes tobacco A patient who has been stabbed A patient with end-stage renal disease

A patient who has been stabbed

Which female patient is at high risk for endometrial cancer? Select all that apply. A patient who is obese A patient with liver cirrhosis A patient who is 20 years old A patient who is on tamoxifen A patient who is on progesterone therapy

A patient who is obese A patient with liver cirrhosis A patient who is on tamoxifen

Which client would have the highest probability of developing breast cancer? A woman whose menarche started at age 15 A woman who went through menopause at age 49 An Asian woman living in Asia A woman whose menarche started at age 9

A woman whose menarche started at age 9

Which statement by Mr. Jones indicates that the discharge teaching by the nurse has been effective? A. "I should carefully monitor and record my weight each day." B. "If I am feeling well, I do not have to take my blood pressure medications regularly." C. "If I have aches and pains, I should take over-the-counter Motrin." D. "I should try to increase the amount of potassium I take in by eating at least one banana every day."

A. "I should carefully monitor and record my weight each day."

The nurse has received the following provider orders for Mr. Jones, who was recently admitted to the medical unit. Which order should the nurse plan to implement first? A. Administer 40 mg furosemide (Lasix) IV daily. B. Implement fluid restriction less than 1,500 mL/24 hr. C. Monitor urine specific gravity with each void. D. Instruct patient to follow a low-sodium diet.

A. Administer 40 mg furosemide (Lasix) IV daily. (Rationale: though each of the provider orders are appropriate for Mr. Jones, administering thediuretic Lasix is of highest priority to promotedincreased excretion of potassium and fluid in order to lower the blood pressure, heart rate, and decrease peripheral edema. The remaining provider orders should be implemented following the administration of Lasix.)

A 25-year-old patient presents to the clinic for her annual gynecological examination. Her last Pap smear was at the age of 21. What should she have today? A. Breast examination, pelvic examination, and Pap smear B. Breast examination, pelvic examination, and Pap smear with HPV cotesting C. Breast examination, pelvic examination, and HPV testing D. Breast and pelvic examinations only

A. Breast examination, pelvic examination, and Pap smear (Rationale: Pap smear should be offered to all women starting at age 21 and then if normal repeated every 3 years until the age of 30. HPV is not indicated as a screening for women between the age of 21 and 30. This patient's last Pap was 4 years ago. She is due for breast, pelvic, and Pap smear now)

The nurse expects to observe which of the following altered laboratory values in a patient diagnosed with renal failure? A. Decreased calcium B. Increased calcium C. Decreased phosphorus D. Increased hematocrit

A. Decreased calcium

The nurse takes a phone call from Ms. Case. Ms. Case is 50 years old and is complaining of a lack of menses, the inability to sleep, hot flashes, and fatigue. What condition is the nurse *not* concerned about? A. Endometriosis B. Pregnancy C. Hypothyroid D. Menopause

A. Endometriosis (Rationale: Endometriosis is usually associated with vaginal bleeding. Pregnancy, hypothyroid, and menopause may all present with amenorrhea, fatigue, and hot flashes. 50-year-old women are unlikely to be pregnant, but it is possible and should be considered.)

The nurse is assessing a patient for a routine physical examination. Which of the following are normal findings when assessing the renal and urinary systems? (Select all that apply.) A. Inability to palpate the kidneys B. CVA tenderness upon percussion C. Absence of a renal artery bruit D. Purulent drainage from the urinary meatus E. Tympanic sounds over an empty bladder upon percussion

A. Inability to palpate the kidneys C. Absence of a renal artery bruit

Mrs. Jones is receiving hospice care in the nursing home. During the assessment, the nurse observes the patient is unconscious and has wet, noisy respirations and cool, mottled extremities. The nurse understands which of the following actions are indicated? (Select all that apply.) A. Notifying the patient's family B. Requesting an order for an anticholinergic medication C. Notifying the patient's provider D. Performing a sternal rub to assess the patient's response E. Performing a full systems assessment

A. Notifying the patient's family B. Requesting an order for an anticholinergic medication C. Notifying the patient's provider E. Performing a full systems assessment ????

The nurse understands that moral distress may result in which of the following? (Select all that apply.) A. Physical exhaustion B. Disagreements among staff C. Anger at family D. Anger at providers E. Feelings of undeserved power

A. Physical exhaustion C. Anger at family D. Anger at providers ALL OF THEM???

The nurse caring for Mr. Jones should implement which of the following priority nursing interventions? (Select all that apply.) A. Strict monitoring of intake and output B. Frequent monitoring of vital signs C. Administer supplemental oxygen D. Safety/fall precautions E. Obtain daily weights

A. Strict monitoring of intake and output B. Frequent monitoring of vital signs D. Safety/fall precautions E. Obtain daily weights

The nurse is caring for a patient receiving IV contrast for a renal CT scan. Nursing responsibilities related to the administration of IV contrast include which of the following? (Select all that apply.) A. To assess for allergies to contrast, iodine, and seafood prior to the study B. To educate the patient to increase fluid intake following the study C. To obtain an order for NPO status following the study D. To ensure that IV contrast is administered only through a central line E. To assess the patient's baseline creatinine value prior to the study

A. To assess for allergies to contrast, iodine, and seafood prior to the study B. To educate the patient to increase fluid intake following the study E. To assess the patient's baseline creatinine value prior to the study

Increased risk to develop breast cancer AFRICAN AMERICAN, AGE 45 or CAUCASIAN, AGE 45

AFRICAN AMERICAN, AGE 45 Rationale: African American women younger than age 50 have a higher risk of developing breast cancer whereas those older than age 50 have a lower risk than Caucasian women of the same age.

Increased risk to develop breast cancer AGE 35, 3 KIDS or AGE 80, 0 KIDS

AGE 80, 0 Kids Rationale: Age is a relative breast cancer risk factor. Women ages 80 to 85 are 15 times more likely to develop breast cancer than women ages 30 to 35. Someone without children is also at a higher risk than a person who has delivered at least one live birth.

Increased risk to develop breast cancer ASIAN PERSON LIVING IN ASIA or ASIAN PERSON LIVING IN THE U.S.

ASIAN PERSON LIVING IN THE U.S. Rationale: Asian females living in Asia have a lower incidence of breast cancer than Asian women who live in the United States. There is no reported etiology for this finding.

The client is experiencing a tracheobronchial obstruction from radiation to the thyroid gland. What assessment change is most concerning? Absence of breath sounds Painful swallowing Hyperkalemia Dependent edema

Absence of breath sounds

When planning care, which nursing diagnoses should the nurse include? Select all that apply. Acute pain Risk for infection Altered elimination Fluid volume deficit Decreased cardiac output

Acute pain Risk for infection Altered elimination (Rationale: Nursing diagnoses that you should include in this patient's plan of care include acute pain related to inflammation and obstruction of kidney, risk for infection related to obstruction, and altered elimination related to obstruction. The other choices are not relevant.)

Tracheobronchial Obstruction Monitor for hyperkalemia Administer prescribed bronchodilators Monitor for hypoxemia Assess for dependent edema Assess for absent breath sounds Monitor for diminished chest excursion Administer prescribed potassium supplements

Administer prescribed bronchodilators Monitor for hypoxemia Assess for absent breath sounds Monitor for diminished chest excursion Rationale: Tracheobronchial obstruction is an oncologic emergency associated with lung cancer. When providing care for a client with this issue, the nurse should immediately administer prescribed bronchodilators and monitor for hypoxemia. In severe and late state obstruction, breath sounds are absent and chest excursion is reduced. Dependent edema and hyperkalemia are not anticipated clinical manifestations associated with this medical emergency.

Risk factors associated with the development of breast cancer include which of the following? Select all that apply. Culture Advanced age Women who have had a lower number of ovulatory cycles Women who have never been pregnant Women who had late onset of menarche and early menopause

Advanced age Women who have never been pregnant

A nurse caring for a patient who is undergoing cancer therapy tells him or her, "Remember to cover your head while outside to safeguard against sun damage." Which adverse effect of the therapy is the patient possibly suffering from for which the nurse is cautioning him/her? Nausea/vomiting Diarrhea Thrombosis Alopecia

Alopecia

A patient reports multiple interruptions during urination and dribbling after urination. A digital rectal exam reveals an enlarged prostate and the prostate-specific androgen levels were found to be elevated. Which therapy is indicated for an early improvement of symptoms? Alprostadil Alpha-adrenergic blockers Sildenafil Phosphodiesterase-5 inhibitors

Alpha-adrenergic blockers

A patient is diagnosed with urolithiasis. Which treatment strategies are most likely to aid in the removal or prevention of small stones passing during urination? Select all that apply. Ureteroscopy Alpha-adrenergic blockers Narcotics and antiemetics Percutaneous nephrolithotomy Extracorporeal shock wave lithotripsy

Alpha-adrenergic blockers Extracorporeal shock wave lithotripsy (Rationale: Alpha-adrenergic blockers like tamsulosin relax the musculature of the lower ureter. This aids in the stone passing through the urine during micturation. Extracorporeal shock wave lithotripsy uses shock waves to break down kidney stones. This allows the patient to pass stone fragments during urination)

Complications of Endometriosis Anemia Premature menopause Pelvic scaring Infertility Depression Catamenial pneumothorax

Anemia Pelvic scaring Infertility Catamenial pneumothorax Rationale: The most common complications of endometriosis include pelvic inflammation and scarring resulting in chronic pelvic pain and infertility. Catamenial pneumothorax is a condition where endometrial implants on the lungs result in cyclic pneumothorax associated with the menstrual cycles. Anemia is a result of the heavy bleeding. Endometriosis does not cause premature menopause or depression directly.

A 52-year-old female patient is diagnosed with metastatic bladder cancer, which has spread to her ovaries and fallopian tubes. Which treatment option is most likely to benefit the patient? Fulguration Anterior exenteration Intravesical chemotherapy Colporrhaphy

Anterior exenteration (Rationale: Anterior exenteration involves the removal of the ovaries and fallopian tubes. This procedure is performed if the cancerous cells have spread to these organs)

A patient is diagnosed with urinary urge incontinence from an overactive bladder. Which treatment strategies are most likely to be beneficial to the patient? Select all that apply. Anticholinergics Systemic estrogen Bladder neck suspension Augmentation cystoplasty Alpha-adrenergic blocking agents

Anticholinergics Augmentation cystoplasty (Rationale: Anticholinergics or antispasmodics are used to treat urinary incontinence. They act by blocking nervous stimulation from the parasympathetic nervous system in order to control bladder muscle contractions. Rationale: Augmentation cystoplasty is a surgical intervention used to treat urinary urge incontinence from bladder contractions.)

The primary health-care provider is explaining to the nurse that intracavernosal injection is contraindicated in a patient with erectile dysfunction. Which medication in the prescription of the patient would the nurse suspect to be the reason for contraindication of the intracavernosal injection? Nitrates Alpha blockers Anticoagulants 5-alpha-reductase-inhibitors

Anticoagulants

Pericardial Effusion Assess for increasing respiratory distress Monitor for signs of poor perfusion Assess for hypoactive bowel sounds Assess for pupillary change Monitor for increasing fatigue Monitor for aortic murmur Position in Trendelenburg

Assess for increasing respiratory distress Monitor for signs of poor perfusion Monitor for increasing fatigue Rationale: The nurse should assess vital signs, signs of dyspnea, poor perfusion, and signs of heart failure such as respiratory distress, fatigue, and edema. Assessment of bowel sounds, pupillary change, and aortic murmur is not warranted. Trendelenburg position is not indicated.

Hypercalcemia Monitor for symptoms of delirium Assess urine output Monitor for decreased peripheral perfusion Assess for muscle weakness Treat nausea and constipation Monitor heart rate for bradycardia Assess for hypotension

Assess for muscle weakness Monitor heart rate for bradycardia Monitor for symptoms of delirium Assess urine output Treat nausea and constipation Rationale: The nurse should monitor calcium, phosphorous, and renal function. Assess for symptoms of hypercalcemia such as delirium, somnolence, muscle weakness, fatigue, polyuria, bradycardia, nausea, and constipation. Blood pressure and perfusion are not impacted.

Blood pressure: 160/90 mmHg with abdominal pain and cramping Hypertension as a result of cardiac disease Assess pain on pain scale Administer an NSAID Administer a diuretic Obtain a 12-lead EKG Assess the pulse rate

Assess pain on pain scale Administer an NSAID Assess the pulse rate Rationale: A client who has a clinical diagnosis of endometriosis and presents with hypertension and pain demonstrates a critical presentation of the disease. Although the BP is elevated, it is most likely caused by pain rather than underlying hypertension. Administration of NSAIDs as a pain relief measure would be indicated. A pain scale assessment pulse rate would be a priority.

The nurse is caring for a patient with cancer-associated tumor lysis syndrome. Which nursing assessment is most beneficial for the patient? Assess the patient for increased urine output. Assess the patient for hyperkalemia. Assess the patient for hypertension. Assess the patient for stridor and hypoxemia.

Assess the patient for hyperkalemia.

A patient with lung cancer is diagnosed with superior vena cava syndrome. Which action is most appropriate for this patient? Assessing for signs of fluid overload Assessing for unsteady gait Assessing for signs of renal dysfunction Assessing for signs of respiratory distress

Assessing for signs of respiratory distress

Laparoscopy Assists in visualization of tissue and organ damage Used only to administer antibiotics at infection site Increases blood flow to surgical site area Provides pathway for surgical excision, cauterization, or laser application Is considered a form of open surgery Most common procedure for endometriosis

Assists in visualization of tissue and organ damage Provides pathway for surgical excision, cauterization, or laser application Most common procedure for endometriosis Rationale: Laparoscopy is the most common type of surgical procedure used for the treatment of endometriosis. It helps to both visualize tissue and provide a pathway for surgical excision, cauterization, or laser application. Laparoscopy is considered a minimally invasive surgical procedure and the instruments can be used to administer any type of medication or solution, not just antibiotics.

Mr. Smith's wife is in the terminal stage of chronic obstructive pulmonary disease (COPD). He is very anxious that his wife be comfortable during her last hours. The nurse finds him tearful in the hall outside the room. Which communication should the nurse use first? A. "My aunt had COPD, and she died on a ventilator. Count your blessings that your wife is not on a machine." B. "I can see you're upset, Mr. Smith. Would you like to talk about it?" C. "Is your family here?" D. "Do you think your wife would like a visit from pastoral care?"

B. "I can see you're upset, Mr. Smith. Would you like to talk about it?"

Which statement by the patient indicates that teaching about chemotherapy was effective? A. "I know everyone loses their hair with chemo." B. "I know it is important that I monitor my temperature." C. "I know I should eat whatever I want so that I don't lose weight while on chemo." D. "I understand that I can skip a chemo treatment if I don't feel well."

B. "I know it is important that I monitor my temperature."

Which statement by the patient indicates teaching about tamsulosin (Flomax) has been effective? A. "Will this medicine help my nausea?" B. "So, this medicine will help me pass the stone by relaxing the muscles?" C. "I need to take this medicine so I don't get an infection." D. "This medicine is for my pain."

B. "So, this medicine will help me pass the stone by relaxing the muscles?"

The nurse recognizes which patient is at greatest risk for a UTI? A. A 35-year-old sexually active male B. A 23-year-old sexually active female C. A 50-year-old sexually active female D. An 18-year-old sexually active male

B. A 23-year-old sexually active female

The nurse understands that an abnormal VCUG indicates which of the following? A. The presence of a mass in the bladder B. An increased risk of kidney infection C. Incomplete emptying of the bladder D. Decreased creatinine clearance

B. An increased risk of kidney infection (Rationale: An increased risk of kidney infection due to retrograde flow up the ureters, a VCUG is not looking for masses, incomplete bladder emptying, or renal failure.)

Which of the following is true of BPH? A. It is a form of metastatic cancer. B. Benign prostatic hyperplasia is not a form of cancer. C. Males as young as 25 years old are equally at risk. D. Benign prostatic hyperplasia is a precursor to testicular cancer.

B. Benign prostatic hyperplasia is not a form of cancer.

The nurse understands that an important laboratory assessment parameter for a patient with cancer that has metastasized to bone is which of the following? A. White blood cell count B. Calcium level C. Glucose level D. Sodium/potassium level

B. Calcium level

Ms. Harris has large uterine fibroids that are causing her pain and heavy bleeding. The nurse is educating her on her treatment options. Which option is considered the only definitive treatment for fibroids? A. Uterine artery embolization B. Hysterectomy C. Lupron administration D. Myomectomy

B. Hysterectomy

The nurse is taking care of Ms. Davis, a 19-year-old with significant pelvic pain not associated with her menstrual cycles. After 6 months of oral contraceptive use and NSAIDs, she has had no relief. The nurse understands the next course of treatment indicated includes: A. Narcotics B. Laparoscopy C. Hysterectomy D. Oophorectomy

B. Laparoscopy

The nurse caring for Mr. Jones incorporates which of the following priority nursing diagnoses into the plan of care? A. Risk for falls B. Risk for imbalanced fluid volume C. Impaired urinary elimination D. Readiness for enhanced knowledge

B. Risk for imbalanced fluid volume

The nurse caring for an 86-year-old female patient who presents with nocturnal polyuria incorporates which priority nursing diagnosis into the plan of care? A. Risk for deficient fluid volume B. Risk for sleep deprivation C. Risk for excess fluid volume D. Risk for urge urinary incontinence

B. Risk for sleep deprivation

Cancer is the ______ most common cause of death in the United States. A. first B. second C. third D. fourth

B. Second

The nurse understands that non-muscle cancers affect only: A. Muscle and surrounding fat B. The urothelium, or inner lining of the bladder C. Structures adjacent to the bladder D. The lobes of the prostate

B. The urothelium, or inner lining of the bladder

The nurse understands which population is at increased risk for developing urinary stones? A. Black females living in Florida B. White males living in Georgia C. Black males living in New York D. White females living in Oregon

B. White males living in Georgia

Increased risk to develop breast cancer BENIGN PROLIFERATIVE BREAST DISEASE or COUSIN WITH BREAST CANCER

BENIGN PROLIFERATIVE BREAST DISEASE Rationale: Benign proliferative breast disease is associated with a 70 percent increased risk for developing breast cancer compared to those who have had benign non-proliferative breast disease. Family history beyond a first-degree relative correlates to only a 15 to 20 percent increased risk.

The nurse is conducting a health history interview for a patient who admits to current tobacco use. Which cancers is this patient at risk for developing? Select all that apply. Colon Hepatic Bladder Breast Lung

Bladder Lung

A patient has hematuria, abdominal discomfort, and a distended abdomen. Which condition does the patient most likely have, according to these symptoms? Brain tumor Bladder cancer Cervical cancer Esophageal cancer

Bladder cancer

A patient is diagnosed with pelvic organ prolapse. Which urinary disorder does the nurse expect if the condition is left untreated? Select all that apply. Ureterolithiasis Nephrolithiasis Bladder cancer Urinary incontinence Urinary tract infection

Bladder cancer Urinary tract infection

The nurse is teaching a client with newly diagnosed benign prostatic hyperplasia about potential complications of a transurethral resection of the prostate. What should the nurse include in the education session? Select all that apply. Bleeding in the bladder Urinary infection Bladder stones Kidney damage Impotence

Bleeding in the bladder Urinary infection

What is the reported action of aromatase inhibitors that are used to treat clinical endometriosis? Block synthesis of estrogen locally Increase hormonal levels Stimulate neural pathways Decrease perfusion to the affected area

Block synthesis of estrogen locally

The primary health-care provider is talking with the nursing student about the causes of bladder stones. Which symptoms does the nurse expect to be related to bladder stones? Select all that apply. Blood in the urine Mineral deposits Obstructed urine flow Stretching of bladder muscles Decreased bladder contraction

Blood in the urine Mineral deposits Obstructed urine flow

Which laboratory test helps to evaluate the kidney function? Blood tests Ultrasound test Urodynamic studies Voiding cystourethrogram (VCUG)

Blood tests

Which complication is related to the use of gonadotropin-releasing hormone agonists and aromatase inhibitors in the treatment of endometriosis? Increased likelihood of upper respiratory infections Bone loss Seizures Heart failure

Bone loss

A nurse working in a clinic has received phone calls from several patients. It is a priority for the nurse to suggest immediate follow-up for the female patient complaining of which of the following? A. Urinary frequency at night B. Chest discomfort after spicy meals C. Difficulty eating due to feeling full and bloated D. Pain with the onset of menses

C. Difficulty eating due to feeling full and bloated (early sign of Ovarian CA is satiety & bloating)

The nurse realizes that which clinical manifestation is an expected finding of BPH? A. Increased urinary output B. Swelling of the penis and scrotum C. Frequent nocturia D. Tea-colored urine

C. Frequent nocturia

Ms. Brown has been diagnosed with LCIS. The nurse will help Ms. Brown schedule which procedure? A. Chemotherapy B. Radiation C. Mammogram D. Surgery

C. Mammogram

The nurse understands that the patient receiving chemotherapy prior to definitive surgical excision is receiving which type of treatment? A. Secondary treatment B. Adjuvant therapy C. Neoadjuvant therapy D. Primary treatment

C. Neoadjuvant therapy

The provider orders a basic metabolic panel to be drawn 12 hours after Mr. Jones's admission. When the results are reported, the nurse recognizes that which of the following laboratory values should be reported to the provider most urgently? A. Creatinine 1.9 mg/dL B. Calcium 8.0 mg/dL C. Potassium 5.7 mEq/L D. Phosphorus 4.2 mEq/L

C. Potassium 5.7 mEq/l Rationale: A potassium level of 5.7 mEq/L is indicative of hyperkalemia, and bc of the dangerous cardiac arrhythmias that can result from hyperkalemia this value requires the most urgent intervention. Although a creatinine of 1.9 mg/dL is above the normal range and is increased from Mr. Jones' previous lab results, it does not require as immediate intervention as the potassium level. A calcium level of 8.0 mg/dL indicates hypocalcemia which is not an uncommon finding in a patient with renal insufficiency or renal failure. A phosphorus level of 4.2 mEq/L is within the normal range and therefore requires no intervention.)

Ms. Green is a 16-year-old who has missed 2 days of school each month this year because of painful menses. She has seen her healthcare provider for treatment. The school nurse is reinforcing the prescribed treatment of oral contraceptives and NSAIDs. The nurse teaches Ms. Green to do which of the following? A. Take her OCPs every other day B. Take the NSAIDs at the onset of symptoms and then as needed C. Start NSAIDs 2 days prior to menses and continue on schedule until symptoms resolve D. Stop the oral contraceptives if she has any vaginal bleeding

C. Start NSAIDs 2 days prior to menses and continue on schedule until symptoms resolve (Rationale: NSAIDs should be initiated 2 days prior to menses and continued on schedule until symptoms resolve. The patient should not wait until she has symptoms. Oral contraceptives should be taken daily without fail.)

What is the difference between the cells of a benign tumor and the cells of a malignant tumor? A. The cells of benign tumors are typically aplastic B. The cells of malignant tumors are apoptotic. C. The cells of malignant tumors lack contact inhibition. D. The cells of benign tumors are characterized by uncontrolled cell growth

C. The cells of malignant tumors lack contact inhibition.

Hospice care is not appropriate in which of these circumstances? A. The patient decides to forego curative treatment. B. The patient's prognosis is 3 to 6 months. C. The patient is undergoing experimental chemotherapy. D. The patient is receiving palliative chemotherapy.

C. The patient is undergoing experimental chemotherapy. (This is active curative therapy)

A patient's orders include the placement of a catheter as a temporary treatment of his BPH. The nurse understands that rationales for this intervention is which of the following? (Select all that apply.) A. Exact monitoring of intake and output B. Retrieving a sterile sample for urinalysis C. To evaluate PVR volume D. To facilitate complete emptying of the bladder E. To avoid unnecessary surgical procedures

C. To evaluate PVR volume D. To facilitate complete emptying of the bladder

Increased risk to develop breast cancer HISPANIC LIVING IN NORTH AMERICA or CAUCASIAN LIVING IN NORTH AMERICA

CAUCASIAN LIVING IN NORTH AMERICA Rationale: Hispanic women in North America have a lower incidence of breast cancer than Caucasian women. There is currently no accepted explanation for this difference.

Increased risk to develop breast cancer CHILDREN AT 25 or CHILDREN AT 35

CHILDREN AT 35 Rationale: Lack of pregnancy and live childbirth before the age of 30 nearly doubles the breast cancer risk.

When teaching a client about irritants that may aggravate urinary incontinence, the nurse should include which substances? Select all that apply. Caffeine Alcohol Gluten Acidic foods Carbonated beverages

Caffeine Alcohol Acidic foods Carbonated beverages

To prevent the potential complication of bone loss associated with gonadotropin-releasing hormone agonists and aromatase inhibitors, what would the nurse expect the healthcare provider to order? Calcium and vitamin D supplementation B-complex vitamin Multivitamin B12 injections

Calcium and vitamin D supplementation

Provide the associated factors in the care of the client with endometriosis. Administration of aromatase inhibitors Can cause bone loss Leads to restoration of normal estrogen levels Prevents osteoporotic fractures from occurring Blocks synthesis of local estrogen Considered experimental based on research evidence Administered as the primary agent in the management of endometriosis Administered in combination with a progestin

Can cause bone loss Blocks synthesis of local estrogen Considered experimental based on research evidence Administered in combination with a progestin Rationale: Evidence-based practice indicates that aromatase inhibitors, which block biosynthesis of estrogen at the local level, be considered for endometriosis treatment. They are administered in conjunction with progestin, a combination oral contraceptive, or GnRH analogue. They are not effective in the treatment of osteoporotic fractures and can cause bone loss. They do not restore normal estrogen levels.

Functional Incontinence Causes frequent urination Causes a loss of urine in inappropriate places Caused by an inability to communicate toileting needs Caused by neurological conditions Caused by an enlarged prostate Implement timed toileting in the plan of care Administer prescribed alpha-adrenergic medications

Causes a loss of urine in inappropriate places Caused by an inability to communicate toileting needs Implement timed toileting in the plan of care (Rationale: A patient who is diagnosed with functional incontinence has an inability to get to the toilet independently or to communicate his or her need to use the toilet. This leads to a loss of urine in inappropriate places. You should include timed toileting in the patient's plan of care. Overflow incontinence is caused by neurological conditions or an enlarged prostate and causes frequent urination. Alpha-adrenergic medications are not used to treat functional incontinence.)

Overflow Incontinence Causes frequent urination Treatment includes the use of antispasmodic medications Caused by spinal cord injury (SCI) Caused by an inability to get to a toilet Causes a strong urge to urinate followed by leakage of urine Causes a loss of urine in inappropriate places Treatment includes the use of alpha-adrenergic medications

Causes frequent urination Caused by spinal cord injury (SCI) Treatment includes the use of alpha-adrenergic medications (Rationale: Overflow incontinence is triggered by a flaccid bladder caused by SCI. Other causes include stroke, diabetes, and neurological diseases. The primary clinical manifestation associated with overflow incontinence is frequent urination. Overflow incontinence is treated with alpha-adrenergic medications. Functional incontinence is caused by an inability to get to a toilet and may lead to loss of urine in inappropriate places. Urge incontinence is associated with a strong urge to urinate followed by the leakage of urine and is treated with antispasmodic medications.)

Which are warning signs of cancer? Select all that apply. Fatigue Alopecia Change in bowel habits Nagging cough or hoarseness Obvious change in a wart or mole

Change in bowel habits Nagging cough or hoarseness Obvious change in a wart or mole

Stress Incontinence Childbirth Caffeine Treated by teaching Kegel exercises Treated by fluid control Occurs with coughing Postmenopausal smoking Artificial sweeteners

Childbirth Treated by teaching Kegel exercises Occurs with coughing Postmenopausal smoking (Rationale: Three primary risk factors for stress incontinence include childbirth, which causes stretching and relaxing of the pelvic floor muscles, ligaments, and urethra; postmenopausal smoking; and obesity. This type of incontinence occurs when abdominal pressure increases with laughing, coughing, lifting, or exercise. Nonsurgical treatment measures for stress incontinence include Kegel exercises. Caffeine and artificial sweeteners are bladder irritants associated with urge incontinence. Fluid control is a nonsurgical treatment measure for urge incontinence.)

The nurse observes decreased serum calcium and hemoglobin, increased blood urea nitrogen (BUN), and creatinine in the laboratory reports of a patient. The patient also has a medical history of uncontrolled diabetes mellitus. Which condition is suspected? Renal cancer Pyelonephritis Acute kidney injury Chronic kidney disease

Chronic kidney disease

While assessing a patient with endometriosis, the nurse finds the patient shows signs of anxiety. Which might be the reason for the patient's condition? Infertility Chronic pelvic pain Inflammation of endometrial implants Scarring of endometrial implants

Chronic pelvic pain

A patient with breast cancer who is undergoing radiation therapy is diagnosed with skin radiodermatitis. Which action will be most beneficial for this patient? Cleaning the area with mild pH-neutral soaps Drying the area under direct sunlight after cleansing Applying lotions, powders, or ointments to the infected area Applying calendula or hyaluronic acid cream to the infected area

Cleaning the area with mild pH-neutral soaps

A PT complains of pain in the mouth 10 days after receiving chemo. Which of the following is a priority? Inadequate nutrional intake Presence of skin breakdown Client complaints of pain Increased susceptibility for infection

Client complaints of pain

The nurse is preparing a client for extracorporeal shock wave lithotripsy. Which assessment finding should be reported to the healthcare provider immediately? Negative urine human chorionic gonadotropin test Client reports taking blood thinner medication daily History of previous stones Creatinine level 0.9 mg/dL

Client reports taking blood thinner medication daily

The nurse is reviewing the patient's reports to determine his or her condition. Which observations would lead to the suspicion of a urinary tract infection? Select all that apply. Cloudy urine Hydronephrosis 2-mm stones in ureters Nitrites in urine cultures 120,000 colony forming units (CFUs) of Escherichia coli

Cloudy urine Nitrites in urine cultures 120,000 colony forming units (CFUs) of Escherichia coli

Which description of pain most likely causes the nurse to suspect a client is suffering from kidney stones? Moderate lower back tightness on one side Mild cramping in lower abdominal area Colicky pain with nausea and vomiting Constant dull pain in bilateral flank areas

Colicky pain with nausea and vomiting ???

While assessing the reports of a patient, the nurse recognizes this situation has occurred. Which surgical intervention is most appropriate for the patient? Colporrhaphy Augmentation cystoplasty Percutaneous nephrolithotomy Extracorporeal shock wave lithotripsy

Colporrhaphy

Which is the diagnostic test of choice for urinary stones? X-ray Computed tomography (CT) scan Cystometrogram Urine culture test

Computed tomography (CT) scan

Which is true regarding urinary tract infections (UTIs)? They are more common in men than in women. The common age group for UTIs in women is 18 to 50 years. Conditions such as pelvic organ prolapse increase the risk of a UTI. Enterobacter is a common intestinal bacterium responsible for 80% of uncomplicated UTIs.

Conditions such as pelvic organ prolapse increase the risk of a UTI.

The nurse is caring for a patient with spinal cord compression. Which symptoms demonstrate late stages? Back pain Weakness, numbness, and tingling Inability to distinguish between hot and cold Constipation or incontinence

Constipation or incontinence

client is receiving a course of chemo for lung CA. Which HOME care instruction should the nurse provide to the PT? Next A bathroom can be shared w/ any member of the family Urinary and bowel excreta is not considered contaminated Disposable plates/plastic utensils must be used during chemo and 2mos after Contaminated linens should be washed separately

Contaminated linens should be washed separately

The nurse is providing client education about the different types of breast biopsy. The nurse describes a procedure that uses a wide needle to remove a large sample. Local anesthesia may or may not be used. What type of biopsy is the nurse describing? Core needle Fine-needle aspiration Vacuum-assisted Surgical excision

Core needle

A patient reports the presence of a painful lump in the breast. The primary healthcare provider uses a needle with a cutter to excise the breast tissue for evaluation. Which diagnostic test was performed in the patient? Core needle biopsy Surgical excision biopsy Vacuum-assisted biopsy Fine needle aspiration cytology

Core needle biopsy

Which detection method is used to make a definitive diagnosis of bladder cancer? Cystoscopy Urine cultures Urine cytology Computed tomography

Cystoscopy

A patient is experiencing ED. He has a history of angina treated with nitroglycerin when necessary. His understanding of his possible medical treatment is indicated by which statement? A. "I guess I'm not bad enough yet to need Viagra." B. "I understand Viagra can make my blood pressure higher." C. "Can Viagra increase my chest pain?" D. "So I guess Viagra is out of the question because it might drop my blood pressure."

D. "So I guess Viagra is out of the question because it might drop my blood pressure."

What is the most specific method of diagnosing a malignancy? A. Serum laboratory tests B. MRI C. CT scan D. Biopsy

D. Biopsy

A patient is complaining of an inability to achieve an erection. The nurse realizes this may be related to a personal medical history of which condition? A. Crohn's disease B. Hepatitis C C. Migraines D. Diabetes

D. Diabetes

The nurse understands that appropriate actions to take after a distressing discussion with a provider over end-of-life issues for a patient include which of the following? A. Avoid communicating with the provider except through the electronic medical record B. Having a drink after work to calm frazzled nerves C. Arguing with colleagues who disagree D. Soliciting support from a nurse manager or ethics consultation

D. Soliciting support from a nurse manager or ethics consultation

The patient with stress incontinence is prescribed Kegel exercises. The nurse tells the patient that Kegel exercises will help: A. Strengthen the detrusor muscle B. Strengthen the posterior urethral valves C. Strengthen the internal sphincter D. Strengthen the external sphincter

D. Strengthen the external sphincter

The nurse asks the patient and family members if they have any questions about the patient's diagnosis and plan of care. This is an example of which PC domain? A. Physical aspects of care B. Psychological and emotional aspects of care C. Ethical and legal aspects of care D. Structure and process of care

D. Structure and process of care

The nurse is caring for an older adult with intermittent urinary incontinence. The nurse recognizes that the incontinence is a result of which structure failing to contract to keep urine in the bladder until micturition? A. The detrusor muscle B. The external urethral sphincter C. The urethra D. The internal urethral sphincter

D. The internal urethral sphincter

The nurse is most concerned about Ms. James, a 60-year-old female patient, when she calls complaining of which of the following? A. Vaginal itching B. A foul vaginal discharge C. Hot flashes at night D. Vaginal bleeding

D. Vaginal bleeding (Concern of endometrial/uterine CA)

A nurse is assessing the vital signs of a patient diagnosed with stones in his or her ureters. Which vital sign change is most likely to be observed? Decreased heart rate Decreased temperature Decreased blood pressure Decreased respiratory rate

Decreased blood pressure (Rationale: A pt w/ urolithiasis will have decreased BP bc of infection & sepsis, dehydration, & complication of untreated stones)

Which clinical finding is present in a patient undergoing chemotherapy for breast cancer? Increased hematocrit Increased hemoglobin Decreased liver enzymes Decreased white blood cell count

Decreased white blood cell count

A patient is diagnosed with stage T2 bladder cancer. The tumor has not metastasized. Which part of the urinary tract is infected according to the tumor, node, metastasis (TNM) staging system? Kidney Urothelium Renal artery Detrusor muscle

Detrusor muscle (Rationale: Stage T2 indicates that the tumor is an invasive cancer that has extended into the detrusor muscle)

A patient reports abdominal pain and vomiting during menses. The nurse finds that the patient takes high levels of omega-6 fatty acids. Which nonpharmacological therapy would be beneficial to the patient? Use of vaginal rings Use of an intrauterine device Decreased physical activity Diet rich in omega-3 fatty acids

Diet rich in omega-3 fatty acids

A patient reports frequent and urgent urination. Upon palpation, an enlarged prostate is observed. Which test should be performed to confirm benign prostatic hyperplasia? Urinalysis Digital rectal exam Alpha-fetoprotein levels Prostate-specific androgen (PSA) levels

Digital rectal exam

Which symptoms in a patient indicate benign prostatic hyperplasia (BPH) to the primary health-care provider? Select all that apply. Dribbling once urination is complete Frequent nocturia Erectile dysfunction Swelling and hardness in the scrotum Multiple interruptions during urination

Dribbling once urination is complete Frequent nocturia Multiple interruptions during urination

A RN is reviewing the record of a PT w/ bladder CA. Which risk factor is associated with Bladder CA? African-American female Recorded age of 35 years Occupation of computer analyzer Drinks a pot of coffee every day

Drinks a pot of coffee every day

Which is true regarding the ductal carcinoma in situ? Ductal carcinoma is metastatic. Ductal carcinoma includes invasive lesions. Ductal carcinoma occurs in the lobule where milk is produced. Ductal carcinoma occurs in the lining of the milk ducts.

Ductal carcinoma occurs in the lining of the milk ducts.

A sexually active patient who smokes is at risk for developing which disease condition? Fibroids Endometriosis Dysmenorrhea Uterine cancer

Dysmenorrhea

Which clinical manifestations should the nurse anticipate when providing care to a patient who is diagnosed with lung cancer? Select all that apply. Dyspnea Flank pain Hematuria Weight loss Chronic cough

Dyspnea Weight loss Chronic cough

The RN assist the PT that is post-op mastectomy, in doing which of the following exercises during the first 24 hours? Hand wall climbing Pendulum arm swings Shoulder abduction and external rotation Elbow flexion and extension

Elbow flexion and extension

The nurse is caring for a patient who has undergone a surgical procedure for breast cancer. Which actions made by the nurse are most useful in the prevention of atelectasis? Encouraging verbalization by the patient of any concerns Administering antiemetics to the patient Monitoring the metabolic profile of the patient Encouraging the patient to cough and breathe deeply

Encouraging the patient to cough and breathe deeply

A client with severe endometriosis is to undergo surgical intervention. Which surgery is the client most likely to have? Laparotomy Dilation and curettage Seed implant Endometrial biopsy

Endometrial biopsy

Which condition may occur due to deficiency of 17-β hydroxysteroid dehydrogenase type 2? Fibroids Menopause Ovarian cancer Endometriosis

Endometriosis

Which condition is observed in a patient with endometriosis? Estrogen production is decreased. Cell-mediated immunity is activated. Estradiol concentration is decreased. 17-β hydroxysteroid dehydrogenase type 2 is increased.

Estradiol concentration is decreased. (Rationale: Estradiol is converted to estrogen, and as a result, the concentration of estradiol is decreased)

An older male adult with a history of kidney stones presents to the emergency department with a family member. Which finding in the assessment would most likely alert the nurse to the possibility of a urinary tract infection while waiting on urinalysis results? Family member states client is confused History of benign prostatic hypertrophy Takes alpha-adrenergic blocker daily Voids 100 mL straw-colored urine

Family member states client is confused

Prevention of bone loss caused by medications that lead to hypoestrogenism Female hormone progestin Calcium supplementation Vitamin D Vitamin E Weight-bearing exercise Swimming

Female hormone progestin Calcium supplementation Vitamin D Weight-bearing exercise Rationale: Medications used to treat endometriosis can cause hypoestrogenism. Clients should receive the female hormone progestin, calcium supplementation, and vitamin D as supportive therapies to prevent bone loss. Weight-bearing exercises will also help prevent bone loss.

Which clinical manifestation is observed in a patient with upper ureteral stones? Flank pain Genital pain Urinary urgency Urinary frequency

Flank pain

Which clinical manifestations should the nurse anticipate when providing care to a patient who is diagnosed with kidney cancer? Select all that apply. Dyspnea Flank pain Hematuria Weight gain Chronic cough

Flank pain Hematuria

The nurse is caring for Samantha, a 30-year-old female who presents to the emergency department with symptoms associated with urolithiasis. Which of Samantha's symptoms supports a diagnosis of urolithiasis? Select all that apply. Polyuria Diarrhea Flank pain Nausea and vomiting Abdominal distention Check Answer

Flank pain Nausea and vomiting

A male client diagnosed with kidney stones is unable to void. What procedure should the nurse anticipate next? Nephrostomy tube placement Percutaneous nephrolithotomy Foley catheter insertion Renal ultrasound imaging

Foley catheter insertion

Increased risk to develop breast cancer HIGHER NUMBER OVULATORY CYCLES or LOWER NUMBER OVULATORY CYCLES

HIGHER NUMBER OVULATORY CYCLES Rationale: Women with higher numbers of ovulatory cycles in their lifetime also carry a higher risk of breast cancer.

The nurse is discharging a client with incontinence who will be performing clean intermittent catheterization. What is the best way for the nurse to confirm the client understands the discharge instructions? Have him or her verbalize back what is taught Have the client write out each step of the process Have him or her teach a family member what needs to occur Have him or her provide a return demonstration

Have him or her verbalize back what is taught

The nurse is caring for a male client with bladder distention. Which outcome is the best indication that he may have benign prostatic hyperplasia? The urine is dark amber. He has a high post-void residue as indicated by bladder scanner. The client has discomfort when voiding. The urine has a foul smell and cloudy color.

He has a high post-void residue as indicated by bladder scanner.

A 25-year-old female patient reports severe abdominal cramping along with headache and vomiting during menses. Which treatments would be beneficial for the patient if she wishes to get pregnant in the future? Select all that apply. Heat therapy Intrauterine devices Oral contraceptive pills Nonsteroidal anti-inflammatory drugs (NSAIDs) Gonadotropin releasing hormone agonist therapy (GnRH agonist)

Heat therapy Nonsteroidal anti-inflammatory drugs (NSAIDs)

The nurse is teaching about the care to be provided to the patient who is diagnosed with cancer and is undergoing chemotherapy. Which actions indicate effective understanding? Select all that apply. Helping the patient to develop a nutritional plan Educating the patient about the treatment regimen Advising the patient to avoid pain medications because they may cause ulcers Advising the patient to take antiemetics if he or she experiences nausea or vomiting Discouraging the patient from discussing his or her condition with others

Helping the patient to develop a nutritional plan Educating the patient about the treatment regimen Advising the patient to take antiemetics if he or she experiences nausea or vomiting

The nurse is collecting data from a client with BPH. Which of the following is a late sign of this disorder? Hematuria Nocturia Decreased force of urine stream Difficulty initiating urine stream

Hematuria

Which symptom is common in urinary cancer? Dysuria Hematuria Incontinence Renal colic

Hematuria

Which complication may develop in a patient undergoing radiation therapy for renal cancer? Hernia Bowel obstruction Hemorrhagic cystitis Bleeding from friable tumors

Hemorrhagic cystitis

Which risk factor is associated with the development of endometriosis? Select all that apply. Primary dysmenorrhea History of type 2 diabetes History of bleeding disorders Family history of endometriosis

History of bleeding disorders Family history of endometriosis

The Pap smear reports of a 23-year-old female patient indicates the presence of atypical cells. Which diagnostic testing would be beneficial in the patient? Pap smear test and human papilloma virus (HPV) test Pap smear test every 3 years Human papilloma virus test (HPV) test Pap smear test and human papilloma virus (HPV) test every 5 years

Human papilloma virus test (HPV) test

A 52-year-old patient is undergoing treatment for vaginal dryness, hot flashes, and painful intercourse. Which risk factor in the patient should the nurse monitor to prevent complications in the patient? Anemia Hypertension Lymphedema Renal function

Hypertension

Pt has glomerulonephritis, what is a risk factor for developing acute renal failure? Bradycardia Hypertension Decreased cardiac output Decreased central venous pressure

Hypertension

The nurse monitors fluid overload in a patient with cancer-associated syndrome of inappropriate antidiuretic hormone (SIADH). Which condition is the nurse aiming to prevent in this intervention? Dyspnea Heart block Leukostasis Hypertension

Hypertension

Which assessment finding is priority for the nurse to address after administering an alpha blocker prescribed for a client with kidney stones? Nausea and vomiting Pain verbalized as 6/10 Decreased urinary output Hypotension

Hypotension

A client tells the nurse that she has breast cancer with metastasis. Based on this disclosure, how would the nurse document the client's cancer staging? I II III IV

IV

If diagnostic imaging and biopsy show that breast cancer has extended into the chest wall, how would this be staged? I II III IV

IV

Which drug is used to treat mixed urge and stress incontinence? Ditropan Doxazosin Imipramine Topical estrogen

Imipramine (Rationale: Imipramine is used to treat mixed stress and urge incontinence. - Topical estrogen is used to treat stress incontinence. - Doxazosin is an alpha-adrenergic blocker used to treat stress and reflex incontinence. - Ditropan is an anticholinergic used to treat only urge incontinence.)

A patient with benign prostatic hyperplasia is diagnosed with bladder damage from the outlet obstruction and asks the nurse why that occurred. Which could be the cause for the bladder damage? Urinary stasis Painful urination Mineral deposits in the bladder Incomplete emptying of bladder

Incomplete emptying of bladder Rationale: The incomplete emptying of the bladder may result in stretching of the muscle walls of the bladder, which may cause bladder damage.

A female patient reports urine leakage while laughing and coughing. How should the nurse document this finding? Enuresis Frequency Hesitancy Incontinence

Incontinence

The nurse knows that which statements are true about benign prostatic hyperplasia (BPH)? BPH is a form of metastatic cancer. Men of any age are at risk. BPH is a precursor to bladder cancer. Incontinence is sometimes a result of BPH.

Incontinence is sometimes a result of BPH.

Which physiological substances may contribute to the development of endometriosis? Select all that apply. Increase in aromatase activity Increased systemic production of estrogen Increased prostaglandin activity in response to estrogen Increase in local estrogen levels Increase in 17-B hydroxysteroid dehydrogenase type 2

Increase in aromatase activity Increased prostaglandin activity in response to estrogen Increase in local estrogen levels

A RN is providing dietary instructions to a PT with uric acid renal calculi. The RN should tell the PT they should: Next Include organ meat-type foods in the diet Increase intake of seafood in the diet Increase intake of legumes in the diet Increase intake of cranberries and citrus fruits

Increase intake of legumes in the diet

The laboratory reports of a patient with kidney disease indicate an increase in the blood urea nitrogen (BUN) levels. Which could be the reason behind this condition? Decreased erythropoietin production Decreased gastrointestinal absorption Increased protein intake and breakdown Increased production of nonvolatile acids

Increased protein intake and breakdown

The nurse is caring for a patient who underwent a mastectomy. Which findings in the patient are signs of infection? Decreased pulse rate Increased temperature Increased body weight Decreased blood pressure

Increased temperature

Which topics should the nurse include in the teaching session to decrease the risk for Samantha developing urolithiasis in the future? Select all that apply. Increasing fluid intake Decreasing citrate intake Increasing sodium intake Decreasing oxalate intake Increasing fatty acid intake

Increasing fluid intake Decreasing oxalate intake (Rationale: Patients who have had a kidney stone should increase fluid intake (1 to 2 L/day) and decrease oxalate intake (rhubarb, chocolate, tea, coffee, and nuts). The patient should be taught to increase, not decrease, citrate intake (lemons and lemonade). Decreasing oxalate and increase citrate intake are both effective in decreasing the risk of stone formation. The patient should be taught to decrease sodium intake, as a low-sodium diet can prevent stones by decreasing the risk for dehydration. Fatty acids are not linked to the formation of kidney stones.)

Which warning signs are a part of the CAUTION list of symptoms? Select all that apply. Indigestion or difficulty swallowing. A persistent sore that does not heal. An infection that takes multiple rounds of antibiotics. A change in a wart or mole that are obvious. Hoarseness or nagging cough.

Indigestion or difficulty swallowing. A persistent sore that does not heal. A change in a wart or mole that are obvious. Hoarseness or nagging cough.

A patient visits the primary health-care provider because of painful urination. The digital exam shows the image provided. Which medication is most likely to be prescribed? Tadalafil Sildenafil Indoramin Alprostadil

Indoramin Rationale: Indoramin can be prescribed to treat benign prostatic hyperplasia. This drug decreases the constriction of the urethra and relaxes the smooth muscles.

A client has been diagnosed with benign prostatic hyperplasia and asks the nurse about alternative therapies to surgery. What should the nurse suggest? Select all that apply. Indwelling catheter Saw palmetto Ginkgo biloba African plum Cernilton

Indwelling catheter Saw palmetto African plum Cernilton

A patient reports anorexia along with cloudy and foul-smelling urine. Which condition should the nurse suspect? Renal cancer Renal trauma Infection Glomerulonephritis

Infection

Which associated complications could occur in a client who has endometriosis? Select all that apply. Infertility Painful sex Excessive bleeding in between menses Deep vein thrombosis Nerve pain

Infertility Painful sex

A patient with benign prostatic hyperplasia is on treatment with 5-alpha-reductase inhibitors. Which action of the nurse is most beneficial to the patient? Suggesting the patient perform only light activities Suggesting the patient increase fluid intake in the evening Informing the patient that erectile dysfunction may occur during the therapy Ensuring the patient that over-the-counter drugs can be safely coadministered

Informing the patient that erectile dysfunction may occur during the therapy

A patient diagnosed with erectile dysfunction is considering treatment options. Which nursing interventions are beneficial to the patient? Select all that apply. Instruct the patient regarding medication administration. Assess for cardiovascular disease and testosterone levels. Assess the patient for unstable emotions and performance anxiety. Assure the patient that priapism is not an emergency but a temporary condition. Be judgmental toward the patient's psychiatric issues and medication administration.

Instruct the patient regarding medication administration. Assess for cardiovascular disease and testosterone levels. Assess the patient for unstable emotions and performance anxiety.

The nurse is developing a plan of care for a patient who will be receiving radiation for cancer treatment. Which nursing interventions will be most beneficial for the patient? Select all that apply. Instruct the patient not to void before abdominal radiation. Suggest that the patient wash exposed skin and apply scented moisturizers. Instruct the patient to practice relaxation techniques. Instruct the patient to avoid washing off treatment ink tattoos. Encourage the patient to change position according to his or her comfort during treatment.

Instruct the patient to practice relaxation techniques. Instruct the patient to avoid washing off treatment ink tattoos.

What is true regarding bowel obstruction in a patient with colon cancer? It can happen due to masses in bowel lumen. It can happen due to internal compression. It can happen due to excess antidiuretic hormone. It can happen due to excessive immature white blood cells.

It can happen due to masses in bowel lumen.

A mother of three states she experiences urinary leakage when coughing or sneezing. Which intervention is most appropriate for the nurse to suggest first? Clean intermittent catheterization Anticholinergic medication Restrict oral fluid intake Kegel exercises

Kegel exercises

A patient with dysmenorrhea is receiving treatment with nonsteroidal anti-inflammatory drugs (NSAIDs) and oral contraceptive pills (OCPs). During a follow-up visit after 6 months, the patient still complains of menstrual abdominal cramps. Which test should the primary healthcare provider recommend for this patient? Laparoscopy Pelvic ultrasound Color Doppler studies Surgical dilation and curettage

Laparoscopy

For each heading related to benign prostatic hyperplasia, select the appropriate treatment options from the answer choices. Shrinking or destroying the tissue of the prostate gland Transurethral Needle Ablation Water-Induced Thermotherapy and Transurethral Ethanol Ablation Transurethral Microwave Therapy Transurethral Resection of the Prostate Transurethral Microwave Therapy Open Prostatectomy Transurethral Needle Ablation Transurethral Incision of the Prostate Laser Surgery Prostatic Stents

Laser Surgery Transurethral Needle Ablation Water-Induced Thermotherapy and Transurethral Ethanol Ablation Transurethral Microwave Therapy Rationale: The laser surgery annihilates and/or removes overgrown prostate tissue. The microwave therapy destroys tissue and causes tissue to shrink. The needle ablation uses radio waves to heat and destroy the tissue. The water-induced therapy uses hot water to cause coagulation necrosis of the prostate tissue, whereas the ethanol ablation creates tissue necrosis with injection of ethanol into the arterioles and venules.

Which procedures can be performed transurethrally? Select all that apply. Open prostatectomy Laser surgery Transurethral microwave therapy Transurethral needle ablation Prostatic stents

Laser surgery Transurethral microwave therapy Transurethral needle ablation Prostatic stents

A patient with advanced pancreatic cancer asks the nurse about common metastatic sites. What locations should the nurse include? Select all that apply. Liver Lungs Rectum Omentum Peritoneum

Liver Lungs Peritoneum

The nurse should include which risk factors when teaching about kidney stone prevention? Select all that apply. Living in a warm climate Family history Low salt intake Low protein intake Previous stones

Living in a warm climate Family history Previous stones ???

Which finding is consistent with a clinical diagnosis of in situ breast cancer? Localized Often leads to metastasis, especially if lobular Associated with poor prognosis Does not require surgical intervention

Localized

A client is told that her breast cancer requires removal of the mass and will conserve as much breast tissue as possible. Based on this information, what type of surgical consent would be required? Radical mastectomy Modified radical mastectomy Removal of adhesions Lumpectomy

Lumpectomy

Which is a common site for metastasis that the nurse should include in the plan of care for a patient who is diagnosed with thyroid cancer? Lung Liver Bladder Rectum

Lung

Increased risk to develop breast cancer MENARCHE AT 15 or MENARCHE AT 10

MENARCHE AT 10 Rationale: Women who started menstruating at an earlier age than average (about age 11) are at higher risk to develop breast cancer.

Increased risk to develop breast cancer MENOPAUSE AT 45 or MENOPAUSE AT 55

MENOPAUSE AT 55 Rationale: Women who go through menopause later than the median ages of 50-51 are at higher risk to develop breast cancer.

What is included in screening tests for breast cancer? Select all that apply. Mammography Sentinel node biopsy Magnetic resonance imaging Ultrasound Chest x-ray

Mammography Magnetic resonance imaging

Which procedure involves the removal of breast tissue in patients with breast cancer? Colposcopy Laparoscopy Mastectomy Hysterectomy

Mastectomy

A patient is told they have metastatic cancer and does not understand what that means. Which statements are true pertaining to this? Select all that apply. Metastasis is the propensity for tumors to spread. Metastasis is the defining characteristic of stage IV cancer. Metastasis leads to stage I cancer. Metastasis leads to the growth of primary tumors. Metastasis can either take place by cell-to-cell transfer or through blood.

Metastasis is the propensity for tumors to spread. Metastasis is the defining characteristic of stage IV cancer. Metastasis can either take place by cell-to-cell transfer or through blood.

A patient with a 5 cm tumor in the breast was diagnosed with ductal carcinoma in situ (DCIS). This treatment option was performed. What is the procedure? Modified radical mastectomy Lumpectomy Axillary lymph node removal Breast conservation surgery

Modified radical mastectomy

Spinal Cord Compression Monitor for constipation or incontinence Assess for headache Assess for back pain Monitor for pupil changes Monitor for weakness, numbness, and tingling Place on fall precautions Monitor for seizures

Monitor for constipation or incontinence Place on fall precautions Monitor for weakness, numbness, and tingling Assess for back pain Rationale: With spinal cord compression, the nurse should assess for back pain, weakness, numbness and tingling, unsteady gait, and the loss of ability to distinguish hot and cold. Depending on level of compression, assess for constipation or incontinence. The client should not experience headache, pupil changes, or seizures.

Superior Vena Cava Syndrome Monitor for signs of poor perfusion Monitor for signs of decreased cardiac output Monitor for hypertension Assess for dyspnea Assess for confusion Monitor for edema of the neck, face, and eyes Monitor for bradycardia

Monitor for signs of poor perfusion Monitor for edema of the neck, face, and eyes Assess for confusion Assess for dyspnea Monitor for signs of decreased cardiac output Rationale: Early assessment changes include dyspnea, edema of neck, face and eyes (most severe in the morning), and prominent upper body vasculature. For later symptoms assess for signs of poor perfusion and decreased cardiac output such as confusion, cyanosis, hypotension, tachycardia.

Can you identify the nursing care that is appropriate for clients experiencing complications associated with cancer treatment? Drag and drop the appropriate interventions for each complication or side effect to the corresponding box. Bowel Obstruction Monitor parenteral nutrition Administer enteral nutrition Assess breath sounds Assess for nausea and vomiting Administer prescribed bisphosphonates Assess for hyperactive bowel sounds before Assess for hypoactive bowel sounds after

Monitor parenteral nutrition Assess for nausea and vomiting Assess for hyperactive bowel sounds before Assess for hypoactive bowel sounds after (Rationale: Bowel obstruction is an oncologic emergency that most commonly occurs with cancers that affect the bowel or abdomen, such as colon, pancreatic, ovarian, hepatic, and prostate cancers. Clinical management includes parenteral nutrition. Nursing implications include assessing for nausea and vomiting. Enteral nutrition is not administered. Pharmacologic treatment may include peristaltic stimulants if the bowel obstruction is not complete. Bisphosphonates are prescribed for hypercalcemia, not a bowel obstruction. Bowel sounds are increased prior to the obstruction and diminished or absent after the obstruction.)

Leukostasis Monitor white blood cell count Monitor for decreased urine output Assess for a headache or transient ischemic attack Monitor for bleeding Assess for signs of occluded microcirculation Assess for constipation Monitor for blurred vision Implement seizure precautions

Monitor white blood cell count Assess for a headache or transient ischemic attack Monitor for blurred vision Monitor for decreased urine output Assess for signs of occluded microcirculation Monitor for bleeding Rationale: The nurse needs to be aware of infection, sludging of blood, and thrombosis. This includes monitoring the white blood cell count; and assessing for signs of occluded microcirculation such as blurred vision, headache, transient ischemic attacks, cerebrovascular accidents, dyspnea, poor peripheral perfusion, and oliguria. The nurse should also assess for signs of bleeding.

A nurse is caring for a patient with acute myelocytic leukemia who has been admitted to the oncology department due to leukostasis. What are the most important nursing interventions for this patient? Select all that apply. Monitor white blood cell count. Assess for signs of occluded microcirculation. Monitor calcium, phosphorous, and renal function. Assess for edema of the face and eyes. Assess for signs of bleeding.

Monitor white blood cell count. Assess for signs of occluded microcirculation. Assess for signs of bleeding.

The client is experiencing hypercalcemia from bone metastasis. What symptoms will the nurse anticipate? Select all that apply. Hypotension Muscle weakness Bradycardia Polyuria Prolonged capillary refill

Muscle weakness Bradycardia Polyuria

While assessing the computed tomography scan reports of a patient, the nurse observes the presence of 4-mm-sized stones in the ureteropelvic junction. Which is the priority treatment provided to the patient? Select all that apply. Narcotics Antiemetics Alpha-adrenergic blockers Percutaneous nephrolithotomy Extracorporeal shock wave lithotripsy

Narcotics Antiemetics Alpha-adrenergic blockers

Which medication classifications should you include in Samantha's plan of care based on the current data? Select all that apply. Narcotics Antibiotics Antiemetics Alpha-adrenergic blockers Nonsteroidal anti-inflammatory drugs (NSAIDs)

Narcotics Antiemetics Alpha-adrenergic blockers Nonsteroidal anti-inflammatory drugs (NSAIDs) (Rationale: Narcotics and/or nonsteroidal anti-inflammatory medicine along with antiemetics are utilized during the trial of passage. Alpha-adrenergic blockers such as tamsulosin (Flomax), doxazosin (Cardura), and terazosin (Hytrin) can be used to relax the musculature of the lower ureter and aid in stone passage. Although antibiotics may be required if symptoms of infection are present, the patient is not currently exhibiting symptoms that indicate a need for antibiotic therapy.)

A patient reports severe colicky pain in the abdomen and difficulty urinating. An ultrasound reveals the presence of 11-mm stones in the ureter. This procedure is recommended for the patient. What is it called? Open surgery removal Nephrolithotomy Ureteroscopy Extracorporeal shock wave lithotripsy

Nephrolithotomy (Rationale: A nephrolithotomy is a surgical intervention performed on patients with stones greater than 10 mm)

The client admits an older man with a bladder infection. What additional symptoms could alert the nurse to benign prostatic hyperplasia as the contributing factor? Select all that apply. Polyuria Scrotal swelling Nocturia Bed wetting Incontinence

Nocturia Bed wetting Incontinence

Which diagnostic test is Samantha likely to have prescribed? Ultrasound Non-contrast CT scan Kidney ureter bladder (KUB) x-ray Magnetic resonance imagery (MRI) Check Answer

Non-contrast CT scan (Rationale: A non-contrast CT scan is the diagnostic modality of choice to diagnose urolithiasis. In addition to identifying calculi, CT can assess for obstructive uropathy such as hydronephrosis (swelling of the kidneys that occurs when urine flow is blocked). An MRI is not a diagnostic modality used for the diagnosis of urolithiasis. A KUB x-ray is quick and inexpensive, but does not detect radiolucent uric acid stones. Ultrasound is recommended in children and pregnant females who should avoid radiation exposure.)

A nurse is teaching about carcinogens. Which example indicates a need for further teaching? Human papilloma virus Tobacco Sun exposure Nonionizing radiation

Nonionizing radiation Rationale: nonionizing radiation is relatively low-energy radiation that does not have enough energy to ionize atoms or molecules, unlike ionizing radiations like gamma rays. So, it does not have the ability to cause cancer. Examples of non ionizing radiation include microwaves, radio waves, infrared radiation, visible light, and lasers.Test taking tip: find the incorrect answer.

Which should the nurse include in the teaching plan for a patient who is prescribed external beam radiation therapy in the treatment of cancer? Restricting visitation Not washing off treatment ink tattoos Pre-medicating with diphenhydramine Maintaining intravenous access throughout treatment

Not washing off treatment ink tattoos

A patient is diagnosed with benign prostate hyperplasia (BPH) and failed to respond to alpha-adrenergic receptor blockers. The laboratory reports of the patient revealed that there are bladder stones of 2.5 cm. Which surgical procedure would the nurse expect to be beneficial for the patient? Laser surgery Open prostatectomy Transurethral incision of the prostate (TUIP) Transurethral resection of the prostate (TURP)

Open Prostatectomy Rationale: An open prostatectomy is a surgical procedure that is performed in patients with BPH along with complications such as bladder stones.

For each heading related to benign prostatic hyperplasia, select the appropriate treatment options from the answer choices. Bleeding as a complication Laser Surgery Water-Induced Thermotherapy and Transurethral Ethanol Ablation Open Prostatectomy Transurethral Incision of the Prostate Transurethral Microwave Therapy Transurethral Resection of the Prostate Transurethral Needle Ablation Prostatic Stents

Open Prostatectomy Transurethral Incision of the Prostate Transurethral Needle Ablation Transurethral Microwave Therapy Transurethral Resection of the Prostate Rationale: When the prostate gland is removed, risk for bleeding and infection arises. Often, clients who have these procedures experience bladder irrigation via a three-way catheter until the bleeding decreases.

For each heading related to benign prostatic hyperplasia, select the appropriate treatment options from the answer choices. Removal of the prostate gland Transurethral Incision of the Prostate Laser Surgery Open Prostatectomy Transurethral Microwave Therapy Transurethral Needle Ablation Prostatic Stents Transurethral Resection of the Prostate Water-Induced Thermotherapy and Transurethral Ethanol Ablation

Open Prostatectomy Transurethral Resection of the Prostate Rationale: With these two procedures, the entire prostate gland is removed. In the transurethral incision procedure, two small incisions are made to relieve compression of the urethra. With the laser surgery the prostate tissue is burned away and removed. In the microwave therapy tissue is destroyed and the gland shrinks. With a needle ablation radio waves are passed through needles placed in the prostate gland. With stents a device is placed in the urethra to open the flow of urine, and in water-induced therapy water is used to cause coagulation necrosis of tissue.

A patient is diagnosed with a very large prostate and bladder stones. The primary health-care provider suggests surgery to remove the prostate. Which surgery does the nurse expect the primary health-care provider will advise? Laser surgery Open prostatectomy Transurethral microwave therapy Transurethral resection of the prostate

Open prostatectomy Rationale: An open prostatectomy is performed when the patient is diagnosed with a very large prostate and bladder stones.

A client has suffered a spinal cord injury. Which urinary incontinence does the nurse anticipate the client will experience? Stress Urge Overflow Functional

Overflow

A patient is undergoing treatment for chronic kidney disease (CKD) with dialysis. During a follow-up visit, the nurse observes from the laboratory tests that the patient has developed nephrotoxicity from a prescribed medication. Which medication is responsible for the patient's condition? Meperidine Morphine Penicillin Acetaminophen

Penicillin

When teaching the postmenopausal PT about performing a BSE, the nurse should tell the patient to: Always begin BSE on the right breast first Palpate the breasts before inspection Perform BSE on the same day every month Call the LIP if both breasts are not the same size

Perform BSE on the same day every month

A 45-year-old male patient has undergone surgery for invasive bladder cancer. Which nursing intervention is most appropriate to clear blood clots from the bladder postsurgery? Perform intravesical immunotherapy . Perform continuous bladder irrigation. Record accurate bladder intake and output. Administer intravesical chemotherapy medications.

Perform continuous bladder irrigation. (Rationale: A radial cystoprostatectomy is a surgical treatment for bladder cancer. Continuous bladder irrigation is used after a tumor excision or biopsy to clear blood & blood clots from the bladder and prevent urine obstruction)

A female with incontinence has this device placed. What is this device? Clean intermittent catheter Pessary Artificial sphincter Containment device

Pessary

Which medication is used to decrease discomfort in a patient with a urinary tract infection (UTI)? Doxazosin Ciprofloxacin Phenazopyridium Trimethoprim-sulfamethoxazole

Phenazopyridium (Rationale: Phenazopyridium is a bladder analgesic that can be used to decrease dysuria, frequency, and urgency in a patient with a UTI)

Syndrome of Inappropriate Antidiuretic Syndrome Assess for bradycardia Monitor for hypertension Monitor sodium levels Monitor for dilute urine Place on seizure precautions Limit fluid intake Assess for confusion

Place on seizure precautions Monitor for hypertension Limit fluid intake Monitor sodium levels Assess for confusion Rationale: For SIADH, the nurse should assess for signs of fluid overload such as tachycardia, hypertension, and hyponatremia such as confusion, seizures, and coma. Fluid restriction is important as is the administration of medication as ordered. The urine will be concentrated. Seizure precautions are important because of hyponatremia and cerebral edema.

Which are the immediate postoperative side effects of breast cancer surgery related to immobility? Select all that apply. Pneumonia Dehiscence Lymphedema Shoulder immobility Pulmonary embolism

Pneumonia Pulmonary Embolism

A client is diagnosed with hydronephrosis due to obstructive kidney stones. The nurse should assess for which electrolyte imbalance that might lead to cardiac dysrhythmia? Potassium Calcium Magnesium Sodium

Potassium

A client presents to the clinic with new onset urinary incontinence. While determining treatment options, which of the following is priority? Making sure the client gets to the bathroom Educating about the increasing incidence of incontinence with age Preventing skin breakdown and controlling odor Educating about pelvic floor exercises

Preventing skin breakdown and controlling odor

Which hormone replacement therapy may reduce the risk of ovarian cancer? Estrogen replacement therapy Androgen replacement therapy Progesterone replacement therapy Estrogen and progesterone replacement therapy

Progesterone replacement therapy

Which describes the action of antispasmodics in treating urinary incontinence? Select all that apply. Promote urethral relaxation Promote bladder relaxation Restore moisture of the urethra Promote the action of anticholinergics Block impulses from the parasympathetic nervous system

Promote bladder relaxation Block impulses from the parasympathetic nervous system (Rationale: Antispasmodics help in relaxing and controlling the bladder. Rationale: Antispasmodics block impulses from the parasympathetic nervous system.)

The nurse is presenting at a conference on men's health about prostate cancer. What information should the nurse include? Select all that apply. Prostate cancer may run in families. Prostate cancer progresses very rapidly. Metastasized prostate cancer may be curable. Prostate cancer generally occurs in older males. Symptoms appear immediately after the formation of tumors.

Prostate cancer may run in families. Metastasized prostate cancer may be curable. Prostate cancer generally occurs in older males.

For each heading related to benign prostatic hyperplasia, select the appropriate treatment options from the answer choices. Options for men who are not surgical candidates Transurethral Resection of the Prostate Transurethral Incision of the Prostate Open Prostatectomy Laser Surgery Transurethral Microwave Therapy Transurethral Needle Ablation Prostatic Stents Water-Induced Thermotherapy and Transurethral Ethanol Ablation

Prostatic Stents Rationale: With this nonsurgical procedure, a prostatic stent is placed in the urethra to keep it open. Either metal or plastic stents may be used. Replacement may be required every 4 to 6 weeks. This is not a long-term solution because of increased risk of urinary tract infections.

An older adult male patient is diagnosed with a urinary tract infection (UTI). Which is the most likely reason for this infection? Sexual activity Long urethra Prostatic enlargement Stretching of the bladder

Prostatic enlargement

A client with benign prostatic hyperplasia is not responding to medications and is a high risk for surgery due to a bleeding disorder. Which procedure is the best option for him? Transurethral resection of the prostate Open prostatectomy Laser surgery Prostatic stents

Prostatic stents

A patient with a urinary tract infection also has a history of incontinence. The provider orders a cystourethrogram, which notes the presence of the backflow of urine. Which treatment strategy is most likely to benefit the patient? Administer antispasmodics. Advise to administer a pessary. Advise to perform Kegel exercises. Provide clean intermittent catheterization

Provide clean intermittent catheterization (Rationale: Because the patient has a backflow of urine, intermittent catheterization should be used to provide complete emptying of the bladder and prevent the infection from spreading into the upper urinary system)

A patient with urge incontinence is treated with high doses of antispasmodics. Which action of the nurse is beneficial to the patient? Administer pessaries. Advise the patient to perform reflex voiding. Provide intermittent catheterization. Instruct the patient to avoid fluid intake 1 hour before sleep.

Provide intermittent catheterization. (Rationale: Antispasmodics may cause a paralysis of the detrusor muscles. Because of this condition, urinary retention may occur. In such situations an intermittent catheter is helpful to the patient.)

A patient is prescribed a 10-day course of ciprofloxacin for the treatment of a urinary tract infection (UTI). Which complication is the patient likely to develop if he or she does not complete the course of ciprofloxacin as prescribed? Pyelonephritis Peripheral neuropathy Backflow of infected urine Irreversible renal damage

Pyelonephritis

A client who has been diagnosed with breast cancer is undergoing breast conservation therapy. Which option would be included in this client's treatment plan of care? Reconstruction therapy Incision and drainage Radiation therapy Instillation of radioactive seed implants

Radiation therapy

A patient with a surgical history for prostate cancer reports erectile dysfunction. A diagnosis confirms that the prostate gland and the seminal vesicles were previously removed. Which surgical treatment has the patient likely undergone? Open prostatectomy Radical prostatectomy Open retropubic technique Transurethral needle ablation

Radical prostatectomy

Which surgical treatment is advisable for a female patient with cervical cancer who wants to retain her fertility? Omentectomy Surgical debulking Radical trachelectomy Radical hysterectomy

Radical trachelectomy

Which diagnostic procedure is used to differentiate among solid masses, tumors, and cysts in the kidney? Renal ultrasound Urine cytology Radionuclide isotope Intravenous pyelogram

Renal ultrasound

Which diagnostic test helps to determine renal blood flow? Pyelogram Ultrasound scan Urine cytology testing Renography

Renography (assesses renal blood flow. Radionuclide injected)

What is the priority teaching point when the nurse is discussing narcotic analgesic side effects with a client diagnosed with kidney stones? Lightheadedness Nausea and vomiting Respiratory depression Constipation

Respiratory depression

Which symptom is common with the cancer complication of pericardial effusion? Leukostasis Respiratory distress Hypercalcemia Bowel obstruction

Respiratory distress

The RN should include which of the following info on a written handout about testicular self-examination? Roll the testicle between the thumb and forefinger Perform the exam every other month Perform the exam after a cold shower Expect the exam to be slightly painful

Roll the testicle between the thumb and forefinger

The RN is preparing a poster to promote prevention of cervical CA. The RN should include which recommendation: Use a commercial douche on a daily basis Perform a monthly breast self exam (BSE) Seek treatment promptly for infections of the cervix Use oral contraceptives as a preferred method of birth control

Seek treatment promptly for infections of the cervix

The diagnostic report of a patient reveals the presence of a 2 cm tumor that is suspected to be invasive. Which type of biopsy may be indicated to determine the extent and stage of breast cancer? Sentinel node biopsy Surgical excision biopsy Vacuum-assisted biopsy Fine needle aspiration cytology

Sentinel node biopsy

A patient with breast cancer is undergoing radiation therapy. During the follow-up visit, the patient reports anorexia to the nurse. What laboratory result should the nurse evaluate? Red blood cell count Serum albumin level Liver enzymes Pancreatic enzymes

Serum albumin level

Total hysterectomy with oophorectomy Severe presentation of endometriosis Results from infertility Prevents ovaries from secreting estrogen Consideration of fertility treatments post-surgical intervention Endometriosis is the leading cause Only performed by open incision

Severe presentation of endometriosis Prevents ovaries from secreting estrogen Rationale: For clients who have an extreme form of endometriosis, sometimes removal of both the uterus and ovaries is required to prevent further estrogen secretion. Removal of the uterus alone is not sufficient. Fibroid tumors are the leading cause for a hysterectomy. Hysterectomies can be performed via laparotomy (an open abdominal incision), transvaginal (through the vagina), or laparoscopically (through several small incisions).

A patient reports the presence of a knot in the right breast and the diagnostic report reveals the presence of a 4 cm tumor with no lymph node involvement. Which cancer stage is suspected in the patient based on the diagnostic report? Stage I Stage II Stage III Stage IV

Stage II

A patient reports pain and the presence of a lump in the left breast. If the diagnostic test reveals the presence of a 2-cm tumor, a sentinel node biopsy may be indicated. Which cancer stage is most likely predicted in the patient based on the diagnostic report? Stage I Stage IIA Stage IIB Stage IIIA

Stage IIA (Stage IIA may have spread to axillary lymph nodes requiring sentinel node biopsy)

Which stage of cancer can a nurse expect in a patient suffering from prostate cancer after further tests reveal a secondary tumor in the rectum? Stage I Stage II Stage III Stage IV

Stage IV

Which action should the nurse teach Samantha to perform in order to determine if the kidney stone has passed? Straining the urine Measuring urine output Documenting daily urine pH Monitoring the urine for ketones

Straining the urine (Rationale: You should teach the patient to strain urine to collect the stone so it can be analyzed. Stone analysis will help dictate patient education and prevention of future stones. If the patient requires strict monitoring of input and output, hospital admission would be required along with placement of a Foley catheter. Urine pH may be monitored during the ED visit to determine the presence of acidic urine, which would support the diagnosis of uric acid stones. Urine is monitored for ketones for patients diagnosed with diabetes mellitus, not urolithiasis.)

Which patient's condition can be treated with artificial urinary sphincter surgery? Total incontinence Stress incontinence Reflex incontinence Instability incontinence

Stress incontinence (Rationale: Artificial urinary sphincter surgery is indicated in a patient with stress incontinence.)

Urge Incontinence Strong urge to urinate Treated with systemic estrogen Uncontrolled leakage of urine More common in women Teach patient to eliminate bladder irritants Nicotine Postmenopausal obesity

Strong urge to urinate Uncontrolled leakage of urine Teach patient to eliminate bladder irritants Nicotine (Rationale: Urge incontinence is characterized by a strong urge to urinate followed by uncontrolled leakage of urine. This type of incontinence is often caused by bladder irritants such as nicotine, caffeine, and artificial sweeteners. Eliminating these irritants is the primary treatment modality. Postmenopausal obesity is a cause or risk factor for stress incontinence, which is more common in women and treated with systemic estrogen.)

Mixed Incontinence Strong urge to urinate followed by uncontrolled leakage of urine Occurs with laughing or coughing Causes frequent urination Include timed toileting in care plan Associated with a flaccid, enlarged bladder caused by an enlarged prostate Caused by postmenopausal smoking and obesity

Strong urge to urinate followed by uncontrolled leakage of urine Occurs with laughing or coughing Caused by postmenopausal smoking and obesity (Rationale: Mixed incontinence is a combination of stress and urge incontinence. Clinical manifestations include a strong urge to urinate followed by uncontrolled leakage of urine, along with incontinence that occurs with activities that increase abdominal pressure, such as laughing or coughing. Mixed incontinence has the same causes as stress and urge incontinence, including postmenopausal smoking and obesity. Overflow incontinence is associated with a flaccid, enlarged bladder. Timed toileting is an appropriate intervention for the patient diagnosed with functional incontinence.)

A patient with lung cancer is prescribed a chemotherapeutic agent that is known to cause alopecia. Which nursing action can be beneficial to help promote the self-esteem of this patient? Suggesting the patient use a wig or hat and wear it once hair loss begins. Encouraging the patient to limit social contacts until hair regrowth occurs. Instructing the patient to wash his or her hair gently and comb gently to minimize hair loss. Informing the patient that hair loss will not be permanent and that the hair will grow back.

Suggesting the patient use a wig or hat and wear it once hair loss begins.

When presented with a clinical diagnosis of endometriosis, which treatment option typically provides the best patient outcome? Antibiotic therapy for 6 months' duration Antibiotic therapy based on blood culture results for 10 to 14 days Surgical intervention as opposed to medical management Oral contraceptive therapy to manage symptoms

Surgical intervention as opposed to medical management

perimenopausal female patient reports urine leakage while laughing and coughing. Which medication is most likely to benefit the patient? Trospium Oxybutynin Darifenacin Systemic estrogen

Systemic estrogen (Rationale: Stress incontinence is more common in women; it occurs when abdominal pressure increases, especially during laughing, coughing, or exercising. Patients with this type of incontinence are prescribed systemic estrogen to restore moisture and flexibility of the urethra.)

Which medication helps in reducing the recurrence of breast cancer for a patient undergoing breast conservation therapy? Paclitaxel Tamoxifen 5-fluorouracil Cyclophosphamide

Tamoxifen

The nurse is providing discharge education to Samantha. Which clinical manifestations might necessitate the need for surgical intervention? Select all that apply. The development of renal colic Pain and burning during urination Pain controlled by narcotics A stone that does not pass in 2 to 3 weeks The decreased incidence of nausea and vomiting

The development of renal colic Pain and burning during urination (Rationale: A patient may need surgery if he or she is experiencing renal colic with pain and burning on urination, as these symptoms indicate a urinary tract infection. Pain that is not controlled by narcotics is also an indication for surgical intervention. A stone that does not pass after 4 to 6 weeks and development of nausea and vomiting, not a decreased incidence of these findings, would indicate the need for surgical intervention.)

A postsurgical patient is admitted to a palliative care facility due to physiological complications. The patient develops respiratory distress during the active dying phase. What is the first step that the nurse should take in this situation? The nurse should elevate the head of the bed. The nurse should administer opiates such as morphine. The nurse should surround the patient with familiar sights, sounds, and smells. The nurse should notify the family members since the patient is in end-of-life care.

The nurse should elevate the head of the bed.

The nurse is working with a client who has been diagnosed with endometriosis. Which statement would indicate that additional teaching is needed? Pain is located in surrounding tissues. Inflammation is located in surrounding tissues. The problem is due to endometrial tissue growth within the uterus. It is the most common cause of secondary dysmenorrhea.

The problem is due to endometrial tissue growth within the uterus.

Which is the characteristic of stage I breast cancer? The tumor is 2 cm or less with no lymph node spread and no distant metastasis. The tumor is 2 cm or less with spread to mobile axillary lymph nodes and no distant metastasis. The tumor is 2 cm or less with spread to fixed axillary lymph nodes and no distant metastasis, The tumor is more than 2 cm but less than 5 cm with spread to mobile axillary lymph nodes and no distant metastasis.

The tumor is 2 cm or less with no lymph node spread and no distant metastasis.

The nurse is preparing to teach a client about interventions to help with functional incontinence. Which intervention should be included in the teaching? Timed toileting Take alpha adrenergic medication Clean intermittent catheterization Fluid restriction

Timed toileting

A patient with a urinary tract infection also has a history of incontinence. The physician orders a cystourethrogram. What is the purpose of this exam? To visualize the renal vasculature To assess the bladder for abnormalities including calculi and masses To assess renal blood flow To determine if vesicoureteral reflux is present

To determine if vesicoureteral reflux is present

Topical estrogen has been prescribed for a client experiencing urinary incontinence. The nurse should tell the client that the medication has been prescribed for which purpose? To block impulses in the nervous system to control the bladder To promote anti-cholinergic actions inhibiting parasympathetic activity To promote urethral relaxation to aid with urinary retention To help restore tone of the urethra and vaginal areas

To help restore tone of the urethra and vaginal areas

The patient is undergoing a bronchoscopy to rule out lung cancer. What is the purpose of this? To determine oxygenation levels and provide ventilation To take tissue samples and bronchial washings To clear excess secretions from the lungs To view the alveoli of the lungs for tumors

To take tissue samples and bronchial washings

Which treatment option would be considered for a patient with severe endometriosis? Dilatation and curettage Vaginal hysterectomy leaving the ovaries intact Lysis of adhesions performed via laparoscopic technique Total hysterectomy with oophorectomy

Total hysterectomy with oophorectomy

For each heading related to benign prostatic hyperplasia, select the appropriate treatment options from the answer choices. Options for small or moderately large prostate gland Transurethral Resection of the Prostate Transurethral Incision of the Prostate Open Prostatectomy Laser Surgery Transurethral Microwave Therapy Transurethral Needle Ablation Prostatic Stents Water-Induced Thermotherapy and Transurethral Ethanol Ablation

Transurethral Resection of the Prostate Transurethral Incision of the Prostate Laser Surgery Transurethral Microwave Therapy Rationale: Certain treatments are only effective on smaller or moderately large prostate glands because of the type of procedure and whether the gland is being removed or tissue is being destroyed. The client with a large prostate gland is limited in his procedural options.

For each heading related to benign prostatic hyperplasia, select the appropriate treatment options from the answer choices. Performed with equipment passed through the urethra Transurethral Resection of the Prostate Transurethral Incision of the Prostate Open Prostatectomy Laser Surgery Transurethral Microwave Therapy Transurethral Needle Ablation Prostatic Stents Water-Induced Thermotherapy and Transurethral Ethanol Ablation

Transurethral Resection of the Prostate Transurethral Incision of the Prostate Prostatic Stents Laser Surgery Transurethral Needle Ablation Transurethral Microwave Therapy Water-Induced Thermotherapy and Transurethral Ethanol Ablation Rationale: The most convenient and least invasive method for working with the prostate gland is through the urethra. An open prostatectomy is required when the prostate is very large and the client who has complications such as stones or bladder damage.

Which surgical option is beneficial for a patient with a medium size prostate who has benign prostatic hyperplasia? Laser surgery Open prostatectomy Transurethral incision of the prostate (TUIP) Transurethral resection of the prostate (TURP)

Transurethral incision of the prostate (TUIP) Rationale: Transurethral incision of the prostate (TUIP) is performed if the patient has a small to medium size prostate.

A patient dies postsurgery due to cardiac failure. What step should the nurse adhere to when providing postmortem care? Remove any tubes and catheters from the body before the autopsy is performed. Treat the body with respect, and incorporate any applicable religious or cultural practices. Refrain from including other nursing staff in the process. Refrain from removing any tubes and cleaning soiled areas of the body, if an autopsy is not required.

Treat the body with respect, and incorporate any applicable religious or cultural practices.

Samantha is diagnosed with a urolithiasis smaller than 5 mm in diameter. She is in minimal pain and urinalysis does not indicate infection. What is the first line of treatment? Trial passage Ureteroscopy Percutaneous nephrolithotomy Extracorporeal shock wave lithotripsy

Trial passage (Rationale: Fifty percent of stones less than 5 mm in size will pass spontaneously. For patients with minimal pain and no hydronephrosis or infection, the first line of treatment is an opportunity to pass the stone without intervention. Surgery is the treatment of choice for a patient who presents with a stone larger than 10 mm. Surgical choices are varied and dependent on stone size, location, and urgency of treatment. Ureteroscopy, percutaneous nephrolithotomy, and extracorporeal shock wave lithotripsy are common surgical procedures for the management of stones.)

A pregnant female is assessed for suspected kidney stones. The nurse should prepare the client for which diagnostic test? Computed tomography scan Magnetic resonance imaging Kidneys-ureters-bladder plain film Ultrasound

Ultrasound

Which diagnostic test is most appropriate to determine the presence of urinary incontinence in pregnant women? Ultrasound testing Intravenous pyelogram Voiding cystourethrogram Kidney ureter bladder X-rays

Ultrasound testing

A pregnant patient with calculi in the lower ureter is prescribed tamsulosin for 6 weeks. During a follow-up visit, the patient reports severe intermittent genital pain and vomiting. The laboratory reports reveal that the pain is related to the stones. Which intervention is most likely to benefit the patient? Ureteroscopy Narcotics and antiemetics Other alpha-adrenergic blockers Extracorporeal shock wave lithotripsy

Ureteroscopy (Rationale: Surgical management via ureteroscopy is indicated when medical interventions fail. This includes failure to remove stones even after 4 to 6 weeks of medical therapy)

A client presents to the clinic after experiencing a strong need to urinate followed by uncontrolled leakage of urine. The nurse suspects the client will be diagnosed with which type of urinary incontinence? Stress incontinence Urge incontinence Functional incontinence Overflow incontinence

Urge incontinence

A patient reports frequency in urination leading to awakening at night, a weak stream of urine, and a feeling that the bladder has not completely emptied. Which diagnostic tests are beneficial to confirm the condition? Select all that apply. Urinalysis Ultrasound of the abdomen Digital rectal exam Magnetic resonance imaging Prostate specific androgen (PSA) levels

Urinalysis Digital rectal exam Rationale: A urinalysis can be performed to rule out the presence of infection. Rationale: The symptoms in the client are indicative of benign prostatic hyperplasia (BPH). A digital rectal exam can confirm BPH.

The nurse is caring for a patient who is diagnosed with prostate cancer as evidenced with this image. Which patient symptom will the nurse suspect? Urinary incontinence Weak urinary stream Urinary tract infections Incomplete emptying of the bladder

Urinary incontinence (late sign of prostate CA)

The nurse is teaching a client with newly diagnosed benign prostatic hyperplasia about complications of the condition. What should the nurse include in the instructions? Select all that apply. Urinary retention Urinary tract infection Bladder stones Kidney damage Impotence

Urinary retention Urinary tract infection

A neighbor asks a nurse if he might have an enlarged prostate gland. Which symptoms are consistent with these suspicions? Select all that apply. Urinating several times at night A weak stream of urine Difficulty starting the flow of urine Premature ejaculation Dribbling after urinating

Urinating several times at night A weak stream of urine Difficulty starting the flow of urine Dribbling after urinating

A patient reports an inability to urinate for 3 days. He or she also reports lower abdominal pain and vomiting. Which complication might the patient develop if the condition is left untreated? Select all that apply. Urosepsis Renal abscess Pyelonephritis Hemorrhagic cystitis Irreversible renal damage

Urosepsis Pyelonephritis Irreversible renal damage

A client is diagnosed with a 3-mm right kidney stone. The nurse should plan to include which teaching with the client? Select all that apply. Use of kidney stone strainers Side effects of analgesics Restricting amount of oral fluid intake Changing positions slowly when taking alpha blockers Increasing the number of dairy products in diet

Use of kidney stone strainers Side effects of analgesics ???

A patient reports uncontrolled loss of urine. Which assessments made by the nurse help design a proper treatment plan for the patient? Select all that apply. Dietary intake Voiding history Smoking history Alcohol consumption Reproductive history

Voiding history Reproductive history

Which of the following precautions should the nurse take during the preparation of a chemo IV infusion? Wear gloves and a mask. Wear gloves, masks and eye protection Wear gloves and a gown Wear gloves, masks and a head covering

Wear gloves, masks and eye protection

Nurse provides d/c instructions to Pt w/ mastectomy w/ axillary lymph node dissection. The nurse tells the PT to: Avoid the use of sunscreen when outdoors Cut the cuticles on a regular basis Wear protective gloves when doing the dishes Avoid the use of moisturizing cream on the affected arm

Wear protective gloves when doing the dishes

The nurse is reviewing the results of a urinalysis on a client presenting for urinary incontinence. Which findings on the urinalysis may indicate infection? Select all that apply. Glucose White blood cells Nitrates Ketones Hematuria

White blood cells Nitrates Hematuria

In terms of age as a risk factor for development of breast cancer, which age group would be at increased risk for developing this type of cancer? Women in the perimenopausal period between ages of 35 and 45 Women between the ages of 80 and 85 Women between the ages of 20 and 30 Females who have just gone through menopause, between the ages of 51 and 55

Women between the ages of 80 and 85

Which women are at the lowest risk for breast cancer? Women who have not become pregnant before 30 years of age Women who had early menarche Women with a high number of ovulatory cycles Women whose pregnancy resulted in live childbirth

Women whose pregnancy resulted in live childbirth


Kaugnay na mga set ng pag-aaral

Unit 6. I fell for a Craigslist job scam

View Set

Ch 10: The Formation of Traditional and E-Contracts

View Set

Vocabulary Workshop Level G Units 1 - 8

View Set

CA Life, Health and Accidental Exam Prep 3

View Set

Lesson 5 - Roman Catholic Church in Medieval Europe

View Set

Vocabulary Workshop Level E, Unit 9, Vocabulary Workshop Level E, Unit 15, Vocabulary Workshop Level E, Unit 14, Vocabulary Workshop Level E, Unit 13, Vocabulary Workshop Level E, Unit 12, Vocabulary Workshop Level E, Unit 11, Vocabulary Workshop Lev...

View Set